LSAT and Law School Admissions Forum

Get expert LSAT preparation and law school admissions advice from PowerScore Test Preparation.

 Administrator
PowerScore Staff
  • PowerScore Staff
  • Posts: 8916
  • Joined: Feb 02, 2011
|
#33752
Complete Question Explanation

Flaw in the Reasoning—CE. The correct answer choice is (B)

In this stimulus, the marketing consultant passes judgment on LRG’s advertising campaign. Apparently, the consultant had previously predicted that the campaign would be unpopular with customers, and would not effectively promote LRG’s new products. LRG did not accept the consultant’s advice and, based on the advice of another consultant, went ahead with the campaign. Now, LRG’s new products are selling very poorly and their sales figures are down for the season. Based on this, the original consultant concludes that “the advertising campaign was ill conceived.”

This is a causal argument. Essentially, the consultant has concluded that the advertising campaign caused the poor sales results:

  • Cause ..... ..... ..... ..... ..... ..... Effect

    advertising campaign ..... :arrow: ..... poor sales
However, we cannot be certain that this is the case. Further, we have evidence that there is some larger problem at work. The advertising campaign was related to LRG’s new products. However, the stimulus tells us that the sales are down so far for the season, and not just related to the new products, which admittedly are selling “especially poorly.” Since the advertising campaign applies only to the sales of the new products, and the sales for the season are not necessarily driven solely by the sales of the new products, we need to consider the possibility that some other cause is responsible for the poor numbers, and that it may not be the fault of the advertising campaign, which was not necessarily ill conceived.

The question stem identifies this as a Flaw in the Reasoning question. Our prephrase is that the marketing consultant assumes that the advertising campaign is the cause of LRG’s low sales figures, when there may be some alternate cause for those numbers.

Answer choice (A): This answer choice is inconsistent with the consultant’s argument. If the consultant were to assume that LRG’s sales would have been even lower without the advertising campaign, then the argument would be treating the campaign as a positive, rather than a negative, influence.

Answer choice (B): This is the correct answer choice because it correctly identifies the consultant’s flawed use of causal reasoning, assuming that the campaign is the cause of the poor sales figures, when some other cause may be at work. Here, the answer choice specifically points to the possibility that the alternate cause is a set of economic factors unrelated to the advertising campaign.

Answer choice (C): Here, the answer choice is incorrect because the consultant did not make any comparison between the sales performance of new and established products.

Answer choice (D): The consultant did not reach any conclusion about higher sales of established products, so the argument could not have assumed what caused higher sales of those products.

Answer choice (E): This answer choice describes a flaw in conditional reasoning. However, the consultant’s conclusion did not result from the use of conditional reasoning.
 Jkjones3789
  • Posts: 89
  • Joined: Mar 12, 2014
|
#17551
Hello in this Flaw question I got it down to A and B. I went with A but the answer was B. I identified the conclusion as the last sentence. Please explain why its not A for me. I thought it may have been B but didnt want to dwell on it and run out of time. Thanks
 David Boyle
PowerScore Staff
  • PowerScore Staff
  • Posts: 836
  • Joined: Jun 07, 2013
|
#17556
Jkjones3789 wrote:Hello in this Flaw question I got it down to A and B. I went with A but the answer was B. I identified the conclusion as the last sentence. Please explain why its not A for me. I thought it may have been B but didnt want to dwell on it and run out of time. Thanks
Hello Jkjones 3789,

Answer A is a rather unusual answer, actually, in that the stimulus says basically nothing about the rival's advertising campaign. In any case, answer A is not helpful.
Answer B simply means that there could be an alternate cause for LRG's low sales figures, i.e., economic conditions, rather than advertising strategy.

Hope this helps,
David
 Jkjones3789
  • Posts: 89
  • Joined: Mar 12, 2014
|
#17560
Isn't a referring to the competing consultant's campaign? I took a to mean that just because it was low doesn't mean that the competing consultant's campaign was ill conceived because it could have been lower without it? I see what you mean with B but could you explain why its not A a little better? I think I may have misinterpreted the meaning of the answer choice.
 David Boyle
PowerScore Staff
  • PowerScore Staff
  • Posts: 836
  • Joined: Jun 07, 2013
|
#17561
Jkjones3789 wrote:Isn't a referring to the competing consultant's campaign? I took a to mean that just because it was low doesn't mean that the competing consultant's campaign was ill conceived because it could have been lower without it? I see what you mean with B but could you explain why its not A a little better? I think I may have misinterpreted the meaning of the answer choice.
Hello,

I thought that there may be a trick here, trying to confuse "competing consultant" with "competitor's advertising campaign", which isn't necessarily the same thing. However, "competitor" could refer to the "competing consultant' after all, for all we know, so: answer A may help a tiny bit, in that, although worded in a convoluted manner, it may mean that the speaker assumes (takes for granted), that things could have been the same or better, without the rival consultant's campaign. While that might be true, it is still sort of weak.
Answer B is much stronger, since it presents an alternate cause. And you are always supposed to look for the best answer, of course.

David
 Frank
  • Posts: 16
  • Joined: Apr 30, 2014
|
#18860
Hey guys,

I'm hoping you can explain what makes B right and A wrong. My pre phrase was actually that there is some kind of alternate cause but after I saw A I started to doubt myself and eventually talked myself into taking A. Going over the question I can't really see how A would not weaken the conclusion.
 Lucas Moreau
PowerScore Staff
  • PowerScore Staff
  • Posts: 216
  • Joined: Dec 13, 2012
|
#18867
Hello, Frank,

Ah, your first instinct with the prephrase was correct. An alternate cause is in fact the correct answer. :ras:

The problem with answer choice A is, well, what competitor's advertising campaign? The only competitor here is the competing consultant, who only competes with the speaking marketing consultant. There are not two separate advertising campaigns here...there is one advertising campaign and two separate opinions on that advertising campaign.

I see where you were going with A, but that flaw in the answer choice means that it can't be correct. Answer choice B is an alternate-cause answer choice correctly executed.

Hope that helps,
Lucas Moreau
 Frank
  • Posts: 16
  • Joined: Apr 30, 2014
|
#18870
Hey Lucas,

Wow, that's tricky. So A is sort of a shell game, it tricks you into making an inference that doesn't really exist and has no backing in the stimulus. If A were re-worded so that it said "takes for granted LRG's sales would not have been lower still in the absence latest marketing campaign", would that be better able to weaken a conclusion that the advertising campaign was ill conceived? Is it simply because answer choice A makes the false assumption that it is the competitor's advertising campaign, that it does not attack the conclusion?
 David Boyle
PowerScore Staff
  • PowerScore Staff
  • Posts: 836
  • Joined: Jun 07, 2013
|
#18879
Frank wrote:Hey Lucas,

Wow, that's tricky. So A is sort of a shell game, it tricks you into making an inference that doesn't really exist and has no backing in the stimulus. If A were re-worded so that it said "takes for granted LRG's sales would not have been lower still in the absence latest marketing campaign", would that be better able to weaken a conclusion that the advertising campaign was ill conceived? Is it simply because answer choice A makes the false assumption that it is the competitor's advertising campaign, that it does not attack the conclusion?
Hello Frank,

Re-worded that way, it might help a little maybe. But answer B is still better, since it gives an alternate cause. The correct answer does not need to assume, as your reworded answer A does, that sales wouldn't've been lower without the latest marketing campaign (which can be rephrased as "sales would've been lower with the latest marketing campaign"). The stimulus also complains that the latest line of products from LRG sold badly. So, even if general sales weren't worse because of the latest campaign, the other problem, i.e., the latest products selling badly, still could've existed and caused problems. ...Also, even if the campaign was unsuccessful, that doesn't mean it was ill-conceived. Other problems might've happened too, even with a well-conceived plan.
So even if your reworded version of answer A might weaken the stimulus a tiny bit, answer B is just much better.

Hope this helps,
David
 mokkyukkyu
  • Posts: 97
  • Joined: Aug 17, 2016
|
#29158
Hi,

First of all...what does "the ad campaign was ill conceived" mean? They did not receive good advice?

I was not sure why D and E are wrong.
For D, I thought it could be right because in the stimulus it says
effective ad→higer sale and D is the contrapositive;
Contrapositive
not higer sale→not effective ad
So I thought it could be right...
For E, I thought it says
"higer sale→effective ad is right" and what the stimulus says (effective ad→higher sale) is wrong

In bothe cases, they think the stimulus says
effective ad→higher sale and think there are some problems in thinking so...

Why are E and D wrong?

Get the most out of your LSAT Prep Plus subscription.

Analyze and track your performance with our Testing and Analytics Package.